Intersection nested, closed sequence of intervals

Click For Summary
The discussion centers on proving the limit of the lengths of a sequence of closed nested intervals exists and demonstrates that if this limit is greater than zero, the intersection of these intervals forms a closed interval of that length. The existence of the limit is established by showing that the sequences of endpoints are convergent. The main challenge lies in proving that if the interval length is positive, the intersection equals the closed interval defined by the limit points. Participants clarify that understanding the definition of intersection helps in proving that points outside the interval cannot be in the intersection, while points within the interval must be included. The conversation concludes with a consensus on the reasoning behind the proof.
mahler1
Messages
217
Reaction score
0
Homework Statement .
Let ##\{I_n\}_{n \in \mathbb N}## be a sequence of closed nested intervals and for each ##n \in \mathbb N## let ##\alpha_n## be the length of ##I_n##.
Prove that ##lim_{n \to \infty}\alpha_n## exists and prove that if ##L=lim_{n \to \infty}\alpha_n>0##, then ##\bigcap_{n \in \mathbb N} I_n## is a closed interval of length ##L##.

The attempt at a solution.

I didn't have problems to prove the existence of the limit: if ##I_n=[a_n,b_n]##, then ##\alpha_n=b_n-a_n## and ##\{a_n\}_n, \{b_n\}_n## are increasing and decreasing bounded sequences respectively, so both are convergent ##\implies \{\alpha_n\}_n## is convergent and ##lim_{n \to \infty} \alpha_n=lim_{n \to \infty}b_n-a_n=lim_{n \to \infty}b_n-lim_{n \to \infty}a_n=b-a##.

Now, I would like to say that if ##b-a>0 \implies \bigcap_{n \in \mathbb N} I_n=[a,b]##, but I don't know hot to prove this part.
 
Physics news on Phys.org
Try showing if x isn't in [a,b] then x isn't in the intersection and if x is in [a,b] then x is in the intersection.
 
Last edited:
  • Like
Likes 1 person
mahler1 said:
Homework Statement .
Let ##\{I_n\}_{n \in \mathbb N}## be a sequence of closed nested intervals and for each ##n \in \mathbb N## let ##\alpha_n## be the length of ##I_n##.
Prove that ##lim_{n \to \infty}\alpha_n## exists and prove that if ##L=lim_{n \to \infty}\alpha_n>0##, then ##\bigcap_{n \in \mathbb N} I_n## is a closed interval of length ##L##.

The attempt at a solution.

I didn't have problems to prove the existence of the limit: if ##I_n=[a_n,b_n]##, then ##\alpha_n=b_n-a_n## and ##\{a_n\}_n, \{b_n\}_n## are increasing and decreasing bounded sequences respectively, so both are convergent ##\implies \{\alpha_n\}_n## is convergent and ##lim_{n \to \infty} \alpha_n=lim_{n \to \infty}b_n-a_n=lim_{n \to \infty}b_n-lim_{n \to \infty}a_n=b-a##.

Now, I would like to say that if ##b-a>0 \implies \bigcap_{n \in \mathbb N} I_n=[a,b]##, but I don't know hot to prove this part.

I guess I'm not seeing anything difficult about this. Can you prove that if x<a then x is not in ##\bigcap_{n \in \mathbb N} I_n##? Same if x>b. And if a<=x<=b then it is? It's just reasoning about what 'intersection' means.
 
  • Like
Likes 1 person
Dick said:
I guess I'm not seeing anything difficult about this. Can you prove that if x<a then x is not in ##\bigcap_{n \in \mathbb N} I_n##? Same if x>b. And if a<=x<=b then it is? It's just reasoning about what 'intersection' means.

Yes, it was just thinking the definition of intersection and apply it, thanks!
 
Question: A clock's minute hand has length 4 and its hour hand has length 3. What is the distance between the tips at the moment when it is increasing most rapidly?(Putnam Exam Question) Answer: Making assumption that both the hands moves at constant angular velocities, the answer is ## \sqrt{7} .## But don't you think this assumption is somewhat doubtful and wrong?

Similar threads

  • · Replies 8 ·
Replies
8
Views
2K
Replies
7
Views
2K
Replies
5
Views
2K
  • · Replies 5 ·
Replies
5
Views
3K
  • · Replies 8 ·
Replies
8
Views
3K
  • · Replies 1 ·
Replies
1
Views
2K
  • · Replies 2 ·
Replies
2
Views
2K
  • · Replies 7 ·
Replies
7
Views
2K
  • · Replies 2 ·
Replies
2
Views
1K
  • · Replies 2 ·
Replies
2
Views
2K